Location via proxy:   [ UP ]  
[Report a bug]   [Manage cookies]                
0% found this document useful (0 votes)
52 views

Lecture - Notes - UECM1034 Chapter4

This document discusses sequences and their limits. [1] A sequence is a list of numbers written in a definite order. [2] The nth term of a sequence is often represented by an. [3] A sequence converges to a limit L if the terms an get arbitrarily close to L as n increases.

Uploaded by

JHAN FAI MOO
Copyright
© © All Rights Reserved
Available Formats
Download as PDF, TXT or read online on Scribd
0% found this document useful (0 votes)
52 views

Lecture - Notes - UECM1034 Chapter4

This document discusses sequences and their limits. [1] A sequence is a list of numbers written in a definite order. [2] The nth term of a sequence is often represented by an. [3] A sequence converges to a limit L if the terms an get arbitrarily close to L as n increases.

Uploaded by

JHAN FAI MOO
Copyright
© © All Rights Reserved
Available Formats
Download as PDF, TXT or read online on Scribd
You are on page 1/ 30

Chapter 4

SEQUENCES, SERIES AND POWER


SERIES

4.1 Sequences
A sequence can be thought of as a list of numbers written in a definite order:

a1 , a2 , a3 , . . . , an , . . .

The number a1 is called the first term, a2 is the second term, and in general an is the nth term.
We will deal exclusively with infinite sequences and so each term an will have a successor an+1 .
Notice that for every positive integer n there is a corresponding number an and so a sequence
can be defined as a function whose domain is the set of positive integers. But we usually write an
instead of the function notation f (n) for the value of the function at the number n.

Notation:
The sequence {a1 , a2 , a3 , . . . , an , . . . } is also denoted by {an } or {an }1
n=1 .

Example 81. Some sequences can be defined by giving a formula for the nth term. Notice that
n doesn’t have to start at 1.
n 1 n
(a) { }n=1 an = { 12 , 23 , 34 , . . . , n+1
n
,...}
n+1 n+1
( 1)n (n + 1) 1 ( 1)n (n + 1)
(b) { n
}n=1 an = n
{ 23 , 39 , 27 4 5
, 81 , . . . , 23 , . . . }
3 3
p p p p p
(c) { n 3}1 n=3 an = n 3, n 3 {0, 1, 2, 3, . . . , n 3, . . . }
p
n⇡ 1 n⇡ 3 1 n⇡
(d) {cos }n=0 an = cos ,n 0 {1, , , 0, . . . , cos ,...} ⇤
6 6 2 2 6

Example 82. Here are some sequences that don’t have a simple defining equation.

(a) The sequence {pn }, where pn is the population of the world as of January 1 in the year n.

85
(b) The Fibonacci sequence {fn } is defined recursively by the conditions

f1 = 1 f2 = 1 fn = fn 1 + fn 2 , n 3

Each term is the sum of the two preceding terms. The first few terms are

{1, 1, 2, 3, 5, 8, 13, 21, . . . } ⇤

Definition 16. A sequence {an } has the limit L and we write

lim an = L or an ! L as n ! 1
n!1

if we can make the terms an as close to L as we like by taking n sufficiently large. If lim an exists,
n!1
we say the sequence converges (or is convergent). Otherwise, we say the sequence diverges (or is
divergent).

Definition 17. A sequence {an } has the limit L and we write

lim an = L or an ! L as n ! 1
n!1

if for every " > 0 there is a corresponding integer N such that

if n > N then |an L| < "

Definition 17 is illustrated below, in which the terms a1 , a2 , a3 , . . . are plotted on a number line.
No matter how small an interval (L ", L + ") is chosen, there exists an N such that all terms of
the sequence from aN +1 onward must lie in the interval.

The points on the graph of {an } must lie between the horizontal lines y = L + " and y = L "
if n > N . The picture must be valid no matter how small " is chosen, but usually a smaller "
requires a larger N .

86
>for integer
Theorem 18. If lim f (x) = L and f (n) = an where n is an integer, then lim an = L.
x!1 n!1
>for real numbers

an an > M

......
1
In particular, since lim = 0 when r > 0, we have
x!1 xr
1 M an M
=

lim =0 if r > 0
n!1 nr

If an becomes large as n become large, we use the notation lim an = 1.


n!1 M >n

Definition 18. lim an = 1 means that for every positive number M there is an integer N such
n!1
that
if n > N then an > M

If lim an = 1, then the sequence {an } is divergent. We say that {an } diverges to 1.
n!1

Theorem 19. If {an } and {bn } are convergent sequences and c is a constant, then
1. lim (an + bn ) = lim an + lim bn
n!1 n!1 n!1

2. lim (an bn ) = lim an lim bn


n!1 n!1 n!1

3. lim can = c lim an , lim c = c


n!1 n!1 n!1

4. lim (an bn ) = ( lim an ) · ( lim bn )


n!1 n!1 n!1

an lim an
5. lim = n!1 if lim bn 6= 0
n!1 bn lim bn n!1
n!1

6. lim apn = [ lim an ]p if p > 0 and an > 0


n!1 n!1

The Squeeze Theorem can also be adapted for sequences as follows:

87
Theorem 20. If an  bn  cn for n n0 and lim an = lim cn = L, then lim bn = L.
n!1 n!1 n!1

Another useful fact about limits of sequences is given by the following theorem.
Theorem
21......

Theorem 21. If lim |an | = 0, then lim an = 0. -


lanks an Ilan1 Un
n!1 n!1
im -1an1 0
=

& nm(an) 0
=

n
Example 83. Find lim . By the Squeeze Theorem,
n!1 n + 1 hm
n- & an 0
=

Solution.

lim nI 1 1
8
N
lim = lim
1 + n1 1
= =

n!1 n + 1
t 10n!1
=

n -
an 1 +
1 +

1
= =1 ⇤
1+0

ln n
Example 84. Calculate lim .
n!1 n

Solution.
Notice that both numerator and denominator approach infinity as n ! 1. We can’t apply
L’Hospital’s Rule directly because it applies not to sequences but to functions of a real variable.
inn h
However,lim 1,m Rule1nx ln x
we can apply L’Hospital’s I'm
to the related
= function f (x) = lim and1obtain0
x0
=

n -> an xx 0x
=

xn - 0 I
=
+ +

ln x 1/x 7
lim = lim =0
x!1 x x!1 1

Therefore, by Theorem 18, we have L'Hospital Rule


ln n
lim =0 ⇤
n!1 n

\( 1(n)
-

Example 85. Determine whether the sequence an = ( 1)n is convergent or divergent.

88
* didn't mention starting number, by default start from 1 *

Solution. an ( 1)"=
-

S(
-1("3of 5( 1)),4-115,(- 1),.......
=91," is read
-
even
If we write out the terms the sequence, we obtain
=

-4,,,...)
{ 1, 1, 1, 1, 1, 1, 1, 1, . . . }

Sincesince the oscillate


the terms term between
an 1 and between
oscillate 1 and
1 infinitely often,infinitely
an does notoften,
approach any number. -

Thus lim
11m
( 1) n
does not exist, that is, the sequence {( 1) n
} is divergent. ⇤
n!1
n-> 0 an does not exist, and the Sanl a
divergent.

( 1)n
Example 86. Evaluate lim if it exists.
n!1 n
Solution. -
11( 1)"-
= 1 +
n>

H((t)" x
+vn>1
( 1)n
lim | | = lim
1
=0
n!1 n n!1 n
alim- =
0 and not=0.
( 1) n By the squeeze Theorem,
Therefore, by theorem,
1M limL 1)4 = 0. ⇤
u- Δ
n!1
-

n
n=0

Theorem 22. If lim an = L and the function f is continuous at L, then lim f (an ) = f (L).
n!1 n!1


Example 87. Find lim sin .
n!1 n
Solution.
Since the sine function is continuous at 0, we see that
Im 0.
=

f(x) = sina is continuous atn 0 =


·

⇡ ⇡
lim sin = sin( lim ) = sin 0 = 0 ⇤
...lm sin 1mn!1f(z) nf nm
=

)] s0
n!1 n =
=
0
=

n!
Example 88. Discuss the convergence of the sequence an = , where n! = 1 · 2 · 3 · · · n.
nn
Solution.
Both numerator and denominator approach infinity as n ! 1, but here we have no corresponding
=,use dt)",as 3),an iti 1)
=with L’Hospital’s Rule (x! is not defined when x is not an integer). Let’s write
function
a. for 1,a 2 33 3)
out a few terms to get a feeling for what happens to an as n get large: 3i -
=
= =
=
=

an =

n 1).a(n)
=

=1
and
1 · 2 opanitll
a2 =
hence,
a3 =
1 · 2 · 3vx>1
an =
1 · 2 · 3···n
1
2·2 3·3·3 n · n · n···n
Note mm" By the squete Theorem, niyan0.
0.
=

that
1 2 · 3···n 1
:
Sanl is a n
convergent
= ( ) and 0 < an 
n n · n···n n
1
We know that ! 0 as n ! 1. Therefore an ! 0 as n ! 1 by the Squeeze Theorem. ⇤
n

89
a 1/ 2
= a 2
=

"

1/4 a 4
"
a a "
y
= =

-
0 a N
-

a3 M
y8
=
a 8
=

y a",a>/
=

Example 89. For what values of r is the sequence {rn } convergent?

Solution. 1 -
7 y ax,0cac/
=

Recall from the graph of the exponential functions that lim a* = 1 for a > 1 and lim ax = 0 for
* from the
x
>x
D M >1 graph
x!1 r x!1
imn putting \ 0, if 0 < r 1
,
0 < a < 1. Therefore, a = r and using
=
theorem, we have
<
⇢ ⑧ ⑥

lim n 1, if r > 1; 0 O
When
1, n-0 rh 1m 1
v =
lim1 r =
0, if 0 < r < 1.
=
=

n!1 0

!
I
>r
W
thatIm
0, lim n
1n ==1 1nelim 00 n = 0. If 1 < r < 0, then 0 < I|r| < 1, so
=

It is obvious and =

n!1 n!1
If I <0<O, then
OCIr1a1, Im frm1 nlmg (r "1 0
-

so =
:

lim |rn | = lim |r|n = 0


:lingv" 0
=

by theorem 21).
n!1 n!1

and Iftherefore
re-1, lim rn = 0 by theorem. If r  1, then {rn } diverges. ⇤
then n!1 Gray diverges.

Remark 12. The sequence {rn } is convergent if 1 < r  1 and divergent for all other values of
r. ⇢
n 0, if 1 < r < 1;
lim r =
n!1 1, if r = 1.

Definition 19. A sequence {an } is called increasing if an < an+1 for all n 1, that is, a1 < a2 <
a3 < . . . . It is called decreasing if an > an+1 for all n 1. It is called monotonic if it is either
increasing or decreasing.

3
Example 90. The sequence { } is decreasing because
n+5 ant -
an

. G
3 3 3
-

an = > = ane
=
3[(n+5) -
(n +

61].- 3
n+5 (n + 1) + 5 n+6
=

(n 6)(n 5)
+ +
(n 1)(n+5)
+

<0

and so an > an+1 for all n 1. ⇤ an-1 -

an <0

an+1< an Un

n
Example 91. Show that the sequence an = is decreasing.
n2 + 1

x 1
Solution. Letf(x) =
Vn>I
Consider the function f (x) = 2 :
(n+1)(1)x +
f(x) =
1
x(2x) -
1 n- 2n1 n-
+ - -

2x2 (n
=
=

(a+1)2
x +1 2
1 x+ 21)2 (n+1)-2
f 0 (x) = = < 0 whenever x >1
(x2 + 1)2 (x2 + 1)2
i. f'(x)<0<>1 -
u- <0(>
-
a > 1E> x>1
Thus f is decreasing on (1, 1) and so f (n) > f (n + 1). Therefore {an } is decreasing. ⇤
it
decreasing
is on (1,0)

an is
decreasing
:.

90
\( 1743
-
m = -
1-an =
1 -1(4-1 M
=

bounded
sequence

Definition 20. A sequence {an } is bounded above if there is a number M such that

an  M for all n 1

It is bounded below if there is a number m such that

m  an for all n 1

If it is bounded above and below, then {an } is a bounded sequence.

[nY "man Innon


= 0
=

Example 92. (a) The sequence 1, 2, 3, . . . , n, . . . has no upper bound since it eventually sur-
passes every M . However, it is bounded below by every real number less than or equal to 1.
The number m = 1 is the greatest lower bound of the sequence.
im an=Imn am" me
1 2 3 n =

(b) The sequence , , , . . . , , . . . is bounded above by every real number greater than or
2 3 4 n+1
equal to 1. The upper bound M = 1 is the least upper bound. The sequence is also bounded
1
below by every number less than or equal to , which is its greatest lower bound. ⇤
2

We know that not every bounded sequence is convergent [for instance, the sequence an = ( 1)n
satisfies 1  an  1 but is divergent] and not every monotonic sequence is convergent (an = n !
1). But if a sequence is both bounded and monotonic, then it must be convergent.

This fact is proved as Theorem 23, but intuitively we can understand why it is true by looking
at figure above. If {an } is increasing and an  M for all n, then the terms are forced to crowd
together and approach some number L.
The proof of Theorem 23 is based on the Completeness Axiom for the set R of real numbers,
which says that if S is a nonempty set of real numbers that has an upper bound M (x  M for
all x 2 S), then S has a least upper bound b. (This means that b is an upper bound for S, but if
M is any other upper bound, then b  M ). The Completeness Axiom is an expression of the fact
that there is no gap or hole in the real number line.

Theorem 23. (Monotonic Sequence Theorem) Every bounded, monotonic sequence is con-
vergent.

91
an Ian+1
Proof. Suppose {an } is an increasing sequence. Since {an } is bounded, the set S = {an |n 1} has
an upper bound. By the Completeness Axiom it has a least upper bound L. Given " > 0, L "
is not an upper bound for S (since L is the least upper bound). Therefore

aN > L " for some integer N

But the sequence is increasing so an aN for every n > N . Thus if n > N , we have

an->L
an > L "
an> aN> L -
2
-
anx -
h
so an > L-G
2 an>,0
-

0L an < " -


anc -
1+ 2

since an  L. Thus 1 -

an<2
-
5cl an <2
-
|L an | < " whenever n > N
so limn!1 an = L. A similar proof (using the greatest lower bound) works if {an } is decreasing.

Example 93. Show that the sequence defined by


1
a1 = 1 an+1 = 3
an
is increasing and an < 3 for all n. Deduce that {an } is convergent and find its limit.

Solution. We show by induction that {an } is increasing and bounded above by 3. Let Pn be the
proposition that an+1 > an and 0 < an < 3. Clearly P1 is true. Assume that Pn is true. Then
1 1 1 1
an+1 > an ) < ) > .
an+1 an an+1 an
1 1
Now an+2 = 3 >3 = an+1 . This proves that {an } is increasing and bounded above by
an+1 an
3, so 1 = a1 < an < 3, that is, {an } is bounded, and hence convergent by the Monotonic Sequence
Theorem. If L = lim an , then lim an+1 = L also, so L must satisfy
n!1 n!1
p
1 2 3± 5
L=3 ) L 3L + 1 = 0 ) L=
L 2
p
3+ 5
But L > 1, so L = . ⇤
2
sequence zen3 52,4,6....?
=

4.2 Series series [(2n) 2 4 6 ....


=
+
+
+

n 1 =

If we try to add the terms of an infinite sequence {an }1


n=1 we get an expression of the form

a1 + a2 + a3 + · · · + an + . . .

92
Example 93 Let P(n):ant1= 3-d is
increasing and and Un>1

when n
1,ac
= = 3 -d 3
=
- 1 2
=
as 2
=
x a,
=

i.a=1c3

:P(I) is
true.

Suppose that PCK) is true for some positive integer K.

..ak =ak+ 1 &ak<3.

ak ak 1 = +

a,4 ak -

1
+

ai -

a +1
a(1 3 + =
-
j, 3
=
-
at +1
an+ 2
=
& an+ 1 3
=
-
i = 3 -
5 <3

iP(k) =>P(k+ 1)

By the Mathematical Induction, the Pln) true Vn>.


we
prove that is

:San3 is
increasing and bounded above

:a, =1 1an <3 0n>, 1

:
Sang is bounded.

By the Monotonic sequence theorem, the convergent.


sequence is

.... him an L =

an 1 3 +
=
-
man+=im Is one (
23 = -
7 ( 3
= u =
-
1 =) (- 32+ 1 0 =

31/3'-4(71)
:2:
2(1)
3IJ
:

+5
Since L>1, so L3 =

him an L. =

showinon an n 2
=

Let m n 1. As + 0,m- &


+
=
n

nea
lim
anti mim
=
am L =
which is called an infinite series (or just a series) and is denoted by the symbol
1
X X
an or an
n=1

We consider the partial sums

s1 = a1
s2 = a1 + a2
s3 = a1 + a2 + a3
s4 = a1 + a2 + a3 + a4

and in general,
n
X
s n = a1 + a2 + a3 + · · · + an = ai
[Sn7:45,5,s,... i=1

These partial sums form a new sequence {sn }, which may or may not have a limit. If lim sn = s
P n!1
exists (as a finite number), we call it the sum of the infinite series an .

P1
Definition 21. Given a series n=1 an = a1 + a2 + a3 + . . . , let sn denote its nth partial sum:
n
X
sn = ai = a1 + a2 + · · · + an
i=1
P
If the sequence {sn } is convergent and lim sn = s exists as a real number, then the series an is
n!1
called convergent and we write
1
X
a1 + a2 + · · · + an + · · · = s or an = s
n=1

The number s is called the sum of the series. Otherwise, the series is called divergent.

1/
n
X
-I'man
P1
Remark 13. Note that n=1 an = lim
n!1
ai . een
=

i=1

Sn

Example 94. An important example of an infinite series is the geometric series


1
X
2 n 1
a + ar + ar + · · · + ar + ··· = arn 1 , a 6= 0
n=1

Each term is obtained from the preceding one by multiplying it by the common ratio r. If r = 1,
then
sn = a + a + · · · + a = na ! ±1. Ifa < 0, Sn na = ->

a <0, Sn = na -> - 8

93
Since limn!1 sn doesn’t exist, the geometric series diverges in this case. If r 6= 1, we have
sn = a + ar + ar2 + · · · + arn 1 ①
rsn = ar + ar2 + · · · + arn 1 + arn ②
① xr
Subtracting these equations, we get

8 -
D
sn rsn = a arn (1
=> -
r)Sn a(1
=
-
V")

a(1 rn )
sn = (4.1)
1 r
If 1 < r < 1, we know that rn ! 0 as n ! 1, so
hm n 0
a(1 rn ) a
n+0
=

lim sn = lim =
n!1 n!1 1 r 1 r
a
Thus when |r| < 1 the geometric series is convergent and its sum is . If r  1 or r > 1, the
1 r
n
sequence {r } is divergent and hence, limn!1 sn does not exist. Therefore the geometric series
diverges in those cases.

We summarize the results as follows:


P
Remark 14. The geometric series 1 n=1 ar
n 1
= a + ar + ar2 + . . . is convergent if |r| < 1 and
its sum is 1
X a
arn 1 = |r| < 1
n=1
1 r
If |r| 1, the geometric series is divergent.

Example 95. Find the sum of the geometric series


10 20 40
5 + + ...
3 9 27
Solution.
The first term is a = 5 and the common ratio is r = 23 . Since |r| = 2
3
< 1, the series is convergent
10/3
sum isr T2
-

and its
15 20 (2)
a5 =
=
i.5
2/3 40
-

5 10 20
/ =
T1 ....
-
= +
- -

5 + + ··· = =3 ⇤ , 3
=

3 9 27 1 ( 23 )

P1
Example 96. Is the series n=1 22n 31 n
convergent or divergent?
Solution.

a 13 OsIMOSIInTlc 414-
Let’s rewrite the nth term of the series in the form arn 1 :
Engin -X 1
3n
2n 1 n
2 3 =
X1
2 n 1 n
(2 ) 3 =
X1
4n
=
X1
4
4( )n 1
3 n 1 3
n=1 n=1 n=1 n=1

We:a 4,rthis
=

recognize 5.
seriesSince
=
v
5 <series
as a geometric 1, with
the aseries is
= 4 and
=

divergent.
r = 4
3
. Since r > 1, the series
diverges. ⇤

94
¯ = 2.31717171... as a ratio integers.
Example 97. Write the number 2.317
Solution.

23
17 17 17
2.317 = 2.3 + ¯0.017 17 17 17
+ = t I ....

2.317 = 2.3 + 3 + 5 + 7 + ...


103
+

105 107
10 10 10
-
After the first term we have a geometric series with a = 103 and r = 1012 . Therefore
17
17

i. 2.317
23
=

t
103
¯
2.317 = 2.3 +
17
103
= 2.3 +
17 a
1000
s
=

/
v
on
=

1 1 1 99
1
-

102 102 100


23 17 1147 a
1147
= = + = ⇤
10 990 495 1 - r
495

P1 1
Example 98. Show that the series n=1 is convergent, and find its sum.
n(n + 1)
Solution.
I
This isLet I
not a geometric series, B back to the definition of 1
A so we go a convergent series and compute
i1
=

k(k 1)
= -

t k
the partial sums,
+

k(k 1) K +

k+ 1
n 1
X
+ · · · + 1(2) 7
1 1 1 When
1 k1 1 =
=

I
A(k 1)
-

i. 1 =
sn =+ Bk = +
+ + 1

i=1
i(i + 1) 1·2 2·3 3·4 n(n + 1)
When K =
0, 1 A(1) =

We can simplify this expression if we use the partial fraction decomposition


A
= = 1
k 2,=

cx t
=
-

5
k -
=

1,1 A(0) B( 1) B =
+
1
E
- =

3,5(x) t
-

1 1 1
=

k =
= -

=
I 1 I i(i + 1) i i+1
.k(k 1)
=
-

k 1
k+
i
+

Thus we have
Enn+1 1,
=

E ti nt]
Xn -
1 X 1 n
1 kn =
-
1,
in n
1
i-1
=
-

n
sn = = ( )
+

i(i + 1) i i+1
= 1 -

i1 i=1 i=1
k n,=
I
=
i n
1 1 1 1 1 1 1
+
-

n(n+1) +1
D = (1 )+( )+( ) + ··· + ( )
emn
I
= 2 2 3 3 4 n n+1
k1 mmA mz 1+1
=

n
=
I
= 1 I 1
=
-

n+1 k= 1k(k1) +
1

1 n+]
="m1 -
10
=
1
=

and so lim sn = lim (1 ) = 1 0 = 1. Therefore the given series is convergent and


:@
1kmkts)
n!1 n!1 n+1
P1 convergent.
is
= 1. ⇤
1 =

n=1
n(n + 1)

Example 99. Show that the harmonic series


X1
1 1 1 1
= 1 + + + + ...
n=1
n 2 3 4

is divergent.

95
Solution. For this particular series it’s convenient to consider the partial sums s2 , s4 , s8 , s16 , s32 , . . .
and show that they become large

s1 = 1
1
s2 = 1 +
2
1 1 1 1 1 1 2
s4 = 1 + +( + )>1+ +( + )=1+
2 3 4 2 4 4 2
1 1 1 1 1 1 1
s8 = 1 + +( + )+( + + + )
2 3 4 5 6 7 8
1 1 1 1 1 1 1
> 1+ +( + )+( + + + )
2 4 4 8 8 8 8
1 1 1 3
= 1+ + + =1+
2 2 2 2
1 1 1 1 1 1 1
s16 = 1 + + ( + ) + ( + ··· + ) + ( + ··· + )
2 3 4 5 8 9 16
1 1 1 1 1 1 1
> 1+ + ( + ) + ( + ··· + ) + ( + ··· + )
2 4 4 8 8 16 16
1 1 1 1 4
= 1+ + + + =1+
2 2 2 2 2
Similarly, s32 > 1 + 52 , s64 > 1 + 62 , and in general

s2n > 1 +
n nIma (1 1)
+
A
=

2
This shows that s2n ! 1 as n ! 1 and so {sn } is divergent. Therefore the harmonic series
diverges. ⇤

P1 En a)
=

+ a2 +... an-1
+ + an
Theorem 24. If the series n=1 an is convergent, then lim an = 0.n!1 Sn-1 a,
=
tac ....
+
an -1
+


①-G: P
Proof. Let sn = a1 + a2 + · · · + an . Then an = s n
sn 1 . Since an is convergent, the sequence
{sn } is convergent. Let limn!1 sn = s. Since n 1 ! 1 as n ! 1, we also have limn!1 sn 1 = s.
Therefore -

lim an = lim (sn sn 1 ) = lim sn lim sn 1 = 0 Letm


1. As n
= -
n- 0,m +a
n!1 n!1 n!1 n!1
:I'mSn 1 -
=

n
Sm S =

5
=
-
3 0
=

P
Remark 15. With any series an wePassociate two sequences: the sequence {sn } of its partial
sums and the sequence {an } terms. If an is convergent, then the limit of sequence {sn } is s (the
sum of the series) and by Theorem the limit of the sequence {an } is 0.
him Sn =

s & I
an 0
=

Remark 16. The converse of Theorem 24 is not true in general. If lim an = 0, we cannot conclude
P P 1 n!1
thatP an is convergent. Observe that for harmonic series n
we have an = n1 ! 0 as n ! 1,
1
but n
is divergent.

96
Theorem 25. (The Test For Divergence) If lim an does not exist or if lim an 6= 0, then the
n!1 n!1
1
X
series an is divergent.
n=1

1
X n2
Example 100. Show that the series diverges.
n=1
5n2 + 4

Solution.
n2
amn,it
Letan
lim aIman
=

n2 1 1
in+4 n = lim = lim = 6= 0 i 0
=

n!1 n!1 5n2 + 4 n!1 5 + 42 5


n
by the Test for Divergence. ⇤
So the
By series
the diverges
Test for Divergence, is divergent. only
P
Remark 17. If we find that lim an 6= 0, we know that an is divergent. If we find that
n!1 P
lim an = 0, we know nothing about the convergence or divergence of an . Remember that
n!1 P
lim an = 0, the series an might converge or it might diverge.
n!1

nm.ninty 0
=

Ininty Innon = 0

P P P
Theorem 26. P If an and bn are
P convergent series, then so are the series 1n=1 can (where c
1 1
is a constant), n=1 (an + bn ), and n=1 (an bn ), and
P1 P1
(i) n=1 can = c n=1 an
P1 P1 P1
(ii) n=1 (an + bn ) = n=1 an + n=1 bn
P1 P P1
(iii) n=1 (an bn ) = 1 n=1 an n=1 bn

1
X 3 1
Example 101. Find the sum of the series ( + n ).
n=1
n(n + 1) 2

Solution.
The series
P 1
From Example 98,
is a geometric series with a = 12 and r = 12 , so niny:
2n
convergent
X1

·
1
1
=t z 1 +
+ ...
-

2 n
=
1
2
1 =1
1
n=1 =
2

:2 (n n+11 in):
P1 D
1
Since = 1, by theorem, the given
+ 2
series is convergent and = n(n +
+

n=1
n(n + 1) n(n 1) +

3 1 + 2n

X1
3(1) X1 X1
13 4. 1
=
+
=
1 1
( + n) = 3 + n
= 3 · 1 + 1 = 4. ⇤
n=1
n(n + 1) 2 n=1
n(n + 1) n=1
2

97
Remark 18. A finite number of terms doesn’t a↵ect the convergence or divergence of a series.
For instance, suppose that we were able to show that the series
1
X n
n=4
n3 +1

is convergent. Since
1
X X n 1
n 1 2 3
= + + +
n=1
n3 + 1 2 9 28 n=4 n3 + 1
1
X n
it follows that the entire series 3
is convergent. Similarly, if it is known that the series
n + 1
P1 n=1
n=N +1 an converges, then the full series

1
X N
X 1
X
an = an + an
n=1 n=1 n=N +1

is also convergent.

4.3 Test of Convergence


In general, it is difficult to find the exact sum of a series. We were able to accomplish this for
P 1
geometric series and the series because in each of those cases we could find a simple
n(n + 1)
formula for the nth partial sum sn . But usually it isn’t easy to compute lim sn . Therefore, in
n!1
the next few sections, we develop several tests that enable us to determine whether a series is
convergent or divergent without explicitly finding its sum.

4.3.1 The Integral Test

THE INTEGRAL TEST


SupposeP1f is continuous, positive, decreasing function on [1, 1) and
R 1 let an = f (n). Then the
series n=1 an is convergent if and only if the improper integral 1 f (x) dx is convergent. In
other words:
R1 P
(a) If 1 f (x) dx is convergent, then 1 n=1 an is convergent.
R1 P
(b) If 1 f (x) dx is divergent, then 1 n=1 an is divergent.

Remark 19. When we use the Integral Test, it is not necessary to start the series or the integral
at n = 1. For instance, in testing the series
X1 Z 1
1 1
2
we use 2
dx
n=4
(n 3) 4 (x 3)

98

Also, it is not necessarily that f be always decreasing. What is important


P1is that f be ultimately
decreasing,
P1 that is, decreasing for x larger than some number N . Then n=N an is convergent, so
n=1 an is convergent.

1
X 1
Example 102. Test the series for convergence or divergence.
n=1
n2 + 1

Solution.
1
The function f (x) = f is continuous, positive, and decreasing on [1, 1) so we use the Integral
Let f(x) x2 +1
a2 +1,n>1.f continuous and
positive, on [1,0) decreasing
=

is
Test:
Z Z
10 f(x) dn =/, t,
dn =

1 with
11m)
1 dx =

emtantal
1 t
1
dx = lim dx = lim tan x]21
1 x2 + 1 t!1 1 x2 + 1 t!1
=
1m [tan 1+

-
tan"
1) = -

=(tan
= lim 1
t

)=
⇡ ⇡
=

t!1 4 2 4 4
Since / flu)du is convergent, by the Integral Test, the series
1
X
R1 1 1
Ent,
Thus is 1convergent
x2 +1
dx is a convergent integral and so, by the Integral Test, the series
n2 +1
is
n=1
convergent. ⇤

X1
1
Example 103. For what values of p is the series p
convergent?
n=1
n

Solution. ① ②
If
1p90, thenhm h=0 1 . If p=8, then him 1 as
If p < 0, then lim p = 1. If p = 0, then lim p = 1. In either case, lim p 6= 0, so the given
n!1 n n!1 n n!1 n
an up.
Let
series diverges Since,
by the anDivergence.
Test for to for pso, by the test for divergence, the
1
If p > 0,
series Ini then isthe function f (x)
divergent, for p-0.
= xp is clearly continuous, positive and decreasing on [1, 1).

Recall that Z 1
If
p20, let flul:he, 1 a>1. So, f is positive, continuous and
decreasing on [1,01.
p
dx converges if p > 1 and diverges if p  1
Recall that I 1 do x is convergent ifp21 P 1 and divergent ifocp.31. By the Integral Test,
It follows from the Integral Test that thewhen
series p converges if p > 1 and diverges if 0 < p  1.
the series [hp is convergent p>I n and divergent for ocp=1.

X1
1
Theorem 27. The p-series is convergent if p > 1 and divergent if p  1.
n=1
np

Example 104. (a) The series

X1
1 1 1 1 1
3
= 3 + 3 + 3 + 3 + ...
n=1
n 1 2 3 4

is convergent because it is a p-series with p = 3 > 1.

99
(b) The series
X1
1 1 1 1 1
1/3
= 1/3 + 1/3 + 1/3 + 1/3 + . . .
n=1
n 1 2 3 4

is divergent because it is a p-series with p = 1


3
< 1. ⇤

Remark 20. We should not infer from the Integral Test that the sum of the series is equal to the
value of the integral. In fact,
X1 Z 1
1 ⇡2 1
= whereas dx = 1
n=1
n2 6 1 x2

Therefore, in general,
1
X Z 1
an 6= f (x) dx
n=1 1

1
X ln n
Example 105. Determine whether the series converges or diverges.
n=1
n

Solution.
ln x
The function f (x) = is positive and continuous for x > 1 because the logarithm function is
x
continuous.Let
Butflu): enc,
it is not a<1.f
obvious whether or not
is f is decreasing,
positive and so we compute
continuous on its(1,0)
derivative:
n(t) ((nx)() 1 1nx 1
· x ln x
-

f(x)
-

f(x)<0(
1=> ln1 x 1nx<0(=> 1(lnx(=e<x
=

22
=
f 0 (x) = x·

= -

x x2 x2
f is decreasing le,N).
Thus f 0 (x) < 0 on
when ln x > 1, that is, x > e. It follows that f is decreasing when x > e and so we
can apply the Integral Test: Let u (n

oflucan-lenuda-linolinnan=nmalunull.nen,"divers
= n
(lnt)
=

0
du= a du
=

Z 1 Z t
ln x ln x (ln x)2 t (ln t)2
dx = lim dx = lim ] = lim =1are
1 x t!1 1 x t!1 2 1 t!1 2
P ln n
Since this improper integral is divergent, the series n
is also divergent by the Integral Test.

4.3.2 The Comparison Test

In the comparison tests the idea is to compare a given series with a series that is known to be
convergent or divergent. For instance, the series
1
X 1
(4.2)
n=1
2n +1

100
P
reminds us of the series 1 n 1 1
n=1 1/2 , which is a geometric series with a = 2 and r = 2 and therefore
is convergent. Because of the series (4.2) is so similar to a convergent series, we have the feeling
that it too must be convergent. Indeed, it is. The inequality
1 1
<
2n + 1 2n
shows that our given series (4.2) has smaller terms than those of the geometric series and therefore
all its partial sum are also smaller than 1 (the sum of the geometric series). This means that its
partial sums form a bounded increasing sequence, which is convergent. It also follows that the
sum of the series is less than the sum of the geometric series:
1
X 1
<1
n=1
2n +1

Similar reasoning can be used to prove the following test, which applies only to series whose terms
are positive.

THE COMPARISONP PTEST


Suppose that an and bn are series with positive terms.
P P
(i) If bn is convergent and an  bn for all n, then an is also convergent.
P P
(ii) If bn is divergent and an bn for all n, then an is also divergent.
Proof. im tn t
=

n
X n
X 1
X
(i) Let sn = ai , t n = bi , t = bn . Since both series have positive terms, the sequences
i=1 i=1 n=1
{sn } and {tn } are increasing (sn+1 = sn + an+1 sn ). Also tn ! t, so tn  t for all n. Since
ai  bi , we have sn  tn . Thus sn  t for all n. This means that {sn } is increasingPand
bounded above and therefore converges by the Monotonic Sequence Theorem. Thus an
converges.
P
(ii) If bn is divergent,Pthen t ! 1 (since {tn } is increasing). But ai bi so sn tn . Thus
sn ! 1. Therefore an diverges. ⇤
1m
= Imtn
In =0
n-
P
In using the Comparison Test we must, of course, have some known series bn for the purpose
of comparison. Most of the time we use one of these series:
P 1
(i) A p-series [ converges if p > 1 and diverges if p  1].
np
P
(ii) A geometric series [ arn 1 converges if |r| < 1 and diverges if |r| 1].

1
X 5
Example 106. Determine whether the series converges or diverges.
n=1
2n2 + 4n + 3

Solution.

101
2n3 4n 372n=
+
+
Vn>,I

Note that 2n4 4n 3+


+
=
(h) fns,1
5 5
5
< 2
+ 4n + 3 2n 2n2
because the left
2n2side
4nhas
vnz,1 zz
3 a bigger denominator. We know that
+ +

22 EI, X1 1
X
Inc is
a 5 5 1
convergent series sincep=c> I
p
=

2
= 2
n=1
2n 2 n=1
n
:It is convergent 1
X
5
is convergent because it’s a constant times a p-series with p = 2 > 1. Therefore
& 2n4n+ 3 isconvergent.
5 2
2n + 4n + 3
By
is convergent. the
⇤ comparison test, n=1
+

n1 =

Remark 21. Although the condition an  bn or an bn in the Comparison Test is given for all
n, we need to verify only that it holds for n N , where N is some fixed integer, because the
convergence of a series is not a↵ected by a finite number of terms.

This is illustrated in the next example.

1
X ln n
Example 107. Test the series for convergence or divergence.
n=1
n

Solution. Note that Inn > lne=1 An>3


Observe that ln n > 1 for n 3 and so
-n
1stVn>3 ln n 1
> n 3
is withn n so it is divergent.
It P
a
p series p=1,
We know that 1/n is divergent (p-series with p = 1). Thus the given series is divergent by the
comparison test, It divergent
is
Comparison
By Test.
the ⇤
direct

Remark 22. The terms of the series being tested must be smaller than those of a convergent series
or larger than those of a divergent series. If the terms are larger than the terms of a convergent
series or smaller than those of a divergent series, then the Comparison Test doesn’t apply. Consider,
for instance, the series ↓direct
X1
1
2 n 1
n=1

The inequality
1 1
>
2n 1 2n
P P
is useless as far as the Comparison Test is concerned because bn = ( 12 )n is convergent and
an > b n .

102
THE LIMITP COMPARISON
P TEST
Suppose that an and bn are series with positive terms. If
an
lim =c >0
n!1 bn

where c is a finite number and c > 0, then either both series converge or both diverge.
an
Proof. Let m and M be positive numbers such that m < c < M . Because is close to c for large
bn
n, there is an integer N such that
an
m< <M when n > N
bn
andPso mbn < an <PM bn when n > N .
P
If
P bn converges, so doesP M bn . Thus an converges by part (i) of the Comparison
P Test. If
bn diverges, so does mbn and part (ii) of the Comparison Test shows that an diverges. ⇤

1
X 1
Example 108. Test the series for convergence or divergence.
n=1
2n 1
Solution.
Letanin)Test1 with an =
We use the Limit Comparison
=

-
and 1
bn= 1inand bn =
and obtain
1 2n 2n
20
manI'mthe a2"
n =
him n
1/(2 27
1) im e2n 1 10 1 =
=

lim = lim n a +

n
1 = lim -

n
= lim =1>0
n!1 bn n!1 1/2 n!1 2 1 n!1 1 1/2n
P since
Since this I
Ibn is convergent it with = 1.
a
1/2n is a convergentgeometricgeometric
is series
=

v
limit exists and series, the given series converges by
=

the By the limit


Limit Comparison
comparison ⇤ IC
Test. test, convergent.
is

X1
2n2 + 3n
Example 109. Determine whether the series p converges or diverges.
5 + n 5
n=1
Solution.
p
2
The dominant part of 2n'the numerator
3n is 2n and the dominant part of the denominator is n5 =

n- z
+

Let an
n5/2 . This suggests taking=

and bn =

2 vn>1
15 n5 +

2
2n + 3n 2n2
an = p bn = 5/2 = 1/2
him an imanctachmentsand is
een n n 20
5 + n5 +

158
=

2 1/2 5/2 3/2 1


an 2n + 3n n 2n + 3n
+

I =C2 n!1isbn
lim = lim p · = lim p
Ibn =

i n!1 + n p -/c
divergent 5since 5 2 =1 n!1 2 5 + n5
2 + n3 2+0
the comparison test, =I amlim q
divergent. = p =1
By n!1
is
2 5 +1 2 0+1
n5
P P 1
Since bn = 2 1/n1/2 is divergent (p-series with p = 2
< 1), the given series diverges by the
Limit Comparison Test. ⇤

P
Notice that in testing many series we find a suitable comparison series bn by keeping only
the highest powers in the numerator and denominator.

103
4.3.3 Alternating Series

An alternating series is a series whose terms are alternatively positive and negative. For example
X 1
1 2 3 4 5 n
+ + + ··· = ( 1)n 1)" bn
-

or 21 -

2 3 4 5 6 n=1
n+1
I( 1jn 1bn
+

I( -
1)" bn
THE ALTERNATING SERIES TEST
If the alternating series
1
X
( 1)n 1 bn = b1 b2 + b3 b4 + . . . bn > 0
n=1

satisfies
(i) bn+1  bn for all n

(ii) lim bn = 0
n!1

then the series is convergent.

Example 110. The alternating harmonic series


1
X ( 1)n 1
1 1 1
1 + + ··· =
2 3 4 n
n=1 Let bx =
satisfies i1 t n int) are
-

ynt1 -
bn =
-
=

1 1
2 (i) bn+1  bn because <
n+1 n
1
I (ii) lim bn = lim =0
n!1 n!1 n

so the series is convergent by the Alternating Series Test. ⇤

X1
( 1)n 3n 3n
Example 111. The series is alternating but An =

4n
n=1
4n 1 -
1

3n 3 3
lim bn = lim = lim 1 = #0
n!1 n!1 4n 1 n!1 4 4
n

so condition (ii) is not satisfied. Instead, we look at the limit of the nth term of the series:

( 1)n 3n
lim an = lim 7
n!1 n!1 4n 1
This limit does not exist, so the series diverges by the Test for Divergence. ⇤

104
1
X
n+1 n2
Example 112. Test the series ( 1) for convergence or divergence.
n=1
n3 + 1

Solution.
Let
The given series is alternating bso
= n
we try to verifyVn> I 1
conditions (i) and (ii) of the Alternating Series
n2
Test. It is not obvious that the sequence given by b = is decreasing. However, if we
the
n
inmn in
n3 + 1
Im b =

n 0n + 1: hm
=
consider the related function f (x) = 3
x2 n find that 0
, we
10
=
=

n3 x +1
Letf(x)
n3 1,vxx,I
=

x33a*
+

x(2 )
2a*
0 2a
(93 1)(2n)+-n" (3x2). f (x) = x(2 xs)
+ -
-

f'(u) 3
(x + 1) 2 =

(n3 1)2
=

(x 1)2
+ (a3 1)2
+
+

f'(x)<0( =>x(2 onlyx3)>0( 0


see that f (x) < 0 if 2 x3 < 0, that is, x > 21/3 .
Since we are considering positive
-
>2 x"<0(:n>,1)
x, we = -

Thusitf is
is decreasing on the interval (21/3 , 1). This means that f (n + 1) < f (n) and therefore
decreasing on (2", 8), so bnll1bn Vn>,2.
bn+1 < bn when n 2. Condition (ii) is readily verified:
n
By the
Alternating Series Test, 21-1)n+ is convergent.
n3 1
1
+

n2 n
lim bn = lim 3 = lim =0
n!1 n!1 n + 1 n!1 1 + 13
n

Thus the given series is convergent by the Alternating Series Test. ⇤

ESTIMATING SUMS
A partial sum sn of any convergent series can be used as an approximation to the total sum s,
but this is not of much use unless we can estimate the accuracy of the approximation. The error
involved in using s ⇡ sn is the remainder Rn = s sn . The next theorem says that for series
that satisfy the conditions of the Alternating Series Test, the size of the error is smaller than bn+1 ,
which is the absolute value of the first neglected term.

P
Theorem 28. (Alternating Series Estimation Theorem) If s = ( 1)n 1 bn is the sum of
an alternating series that satisfies

(i) 0  bn+1  bn and (ii) lim bn = 0 So the series 13


n!1
convergent
then
|Rn | = |s sn |  bn+1

X1
( 1)n
Example 113. Find the sum of the series correct to three decimal places. bn 1
n!
=

n=0 n!

Solution. We first observe that the series is convergent by the Alternating Series Test because
1 1 1
(i) = < bn - bn
(n + 1)! n!(n + 1) n!
+

(ii) 0 <
1 1
< ! 0 so
1
! 0 as n ! 1
Ima bn 0
=

n! n n!
105
To get a feel for how many terms we need to use in our approximation, let’s write out the first few
terms of the series:
1 1 1 1 1 1 1 1 b 7 =0.001389 =

s = + + + + ...
0! 1! 2! 3! 4! 5! 6! 7! 1
b7 5048 =0.0001984 lonwards
=

1 1 1 1 1 1
= 1 1+ + + + ...
2 6 24 120 720 5040
1 1
Notice that b7 = < = 0.0002 and
5040 5000
biz by onwards 1 1 1 1 1
3 decimal
<is 0. s6 = 1 1 + + + ⇡ 0.368056
2 6 24 120 720
By the Alternating Series Estimation Theorem we know that
|s s6 |  b7 < 0.0002
This error of less than 0.0002 does not a↵ect the third decimal place, so we have s ⇡ 0.368 correct
to three decimal places. ⇤

Remark 23. The rule that the error (in using sn to approximate s) is smaller than the first
neglected term is, in general, valid only for alternating series that satisfy the conditions of the
Alternating Series Estimation Theorem. The rule does not apply to other types of series.

4.3.4 Absolute Convergence, The Ratio and The Root Tests


P
Given any series an , we can consider the corresponding series
1
X
|an | = |a1 | + |a2 | + |a3 | + . . .
n=1

whose terms are the absolute values of the terms of the original series.

P P
Definition 22. A series an is called absolutely convergent if the series of absolute values |an |
is convergent.
P
Notice that if an is a series with positive terms, then |an | = an and so absolute convergence
is the same as convergence in this case.

Example 114. The series


Letb n 1
=

1 n2
X ( 1)n 1
1 1 1
=1 + 2 + ... Red bn m r 0
n2 2 2 3 42
=
=

n=1
bnt:I
(n + 1)2 ihc bnfnx,1
=

is absolutely convergent because


X1 X1 So I H2" is convergent by the

( 1)n 1
1 1 1 1 Series
= = 1 + + + + ... Alternating Test.

n=1
n2 n=1
n 2 2 2 3 2 4 2

is a convergent p-series (p = 2). ⇤

P> I 106
Letbe: h nlm bn = 0 & bnt nh ih
=

bn
=

Vn

Example 115. We know that the alternating harmonic series


X 1
( 1)n 1 1 1 1
=1 + + ... is
convergent by the Alternating series

n=1
n 2 3 4
Test.

in convergent, but it is not absolutely convergent because the corresponding series of absolute
values is 1 1
X ( 1)n 1 X 1 1 1 1
| |= = 1 + + + + ...
n=1
n n=1
n 2 3 4
which is the harmonic series (p-series with p = 1) and is therefore divergent. ⇤

P
Definition 23. A series an is called conditionally convergent if it is convergent but not
absolutely convergent.

Example 115 shows that the alternating harmonic series is conditionally convergent. Thus it
is possible for a series to be convergent but not absolutely convergent.

P
Theorem 29. If a series an is absolutely convergent, then it is convergent. -
lan) = an?lanl

Proof. Observe that the inequality 8 - an +1an1 = 21an/

0  an + |an |  2|an |
P P
P because |an | is either an or an . If an is absolutely convergent,
is true P then |an | is convergent,
so 2|an | is convergent. Therefore, by the Comparison Test, (an + |an |) is convergent. Then
X X X
an = (an + |an |) |an |
is the di↵erence of two convergent series and is therefore convergent. ⇤

Example 116. Determine whether the series


X1
cos n cos 1 cos 2 cos 3
2
= 2
+ 2 + 2 + ...
n=1
n 1 2 3
is convergent or divergent.
Solution.
This series has both positive and negative terms, but it is not alternating. We can apply the
Comparison Test to the series of absolute values
cosn
crn
I z SinceX 1 las n/<1 *n>, 1, so
1coon)s
=

1
X Vn>(
·

n2 cos n | cos n|
| 2 |=
n n2
It is a pseries with p= 221, n=1
so it n=1
is
convergent,
Since | cos n|  1 for all n, we have
| cos n| I 1
I cosn
By the  2is
Direct comparison Test, I 2
n n2 n convergent. P
P
We know that 1/n2 is convergent (p-series with p = 2) and therefore | cos n|/n2 is convergent
So, the series [coon
by the Comparison
is
Test. Thusabsolutely convergent
the given series 1
P cos and
n
is
hence convergent
absolutely convergent
by the theorem
and therefore
.

n=1
n2 (theorem 29)
convergent. ⇤

107
THE RATIO TEST Eziest

an+1 P1
(i) If lim | | = L < 1, then the series n=1 an is absolutely convergent (and therefore
n!1 an
convergent).
an+1 an+1 P
(ii) If lim | | = L > 1 or lim | | = 1, then the series 1 n=1 an is divergent.
n!1 an n!1 an

an+1
(iii) If lim | | = 1, the Ratio Test is inconclusive; that is, no conclusion can be drawn about
n!1 an P
the convergence or divergence of an .

an+1
Remark 24. Part (iii) of the Ratio Test says that if lim | | = 1, the test gives no information.
n!1 an
P
For instance, for the convergent series 1/n2 we have
1
an+1 (n + 1)2 n2 1
= = = !1 as n ! 1
an 1 (n + 1) 2 (1 + n1 )2
n2
P
whereas for the divergent series 1/n we have
1
an+1 n 1
= n+1 = = !1 as n ! 1
an 1 n+1 1 + n1
n
P
Therefore, if limn!1 |an+1 /an | = 1, the series an might converge or it might diverge. In this
case the Ratio Test fails and we must use some other test.

1
X n3
Example 117. Test the series ( 1)n for absolute convergence.
n=1
3n

Solution.
Let Test
We use the Ratio 1 a1/6
anwith n 3
n = ( 1) nV
=
-

ns, n
/3n : I

and HIShH(n+s 34 (n + 1)
3
1 1(n
1
34
n +1/3.
lim
+

im ( 1) n+1
ntm 3 (n
-

nD an+1 1 n+1 (n + 1) 3
3n| = | H(+p3 3
an
=
->
+
3n 1
+
| 3 | = L n+1
-

1(u · 3 ns
an n 3 n
( 1)n n
=I "(1th) 3:
G11+0):
1 n + 1a
3
1 1 1
= ( )3 = (1 + )3 ! < 1
3 n 3 n 3
By the Ratio Test, the series an is absolutely convergent
and
Luna convergent.
Thus by the Ratio Test, the given series is absolutely convergent and therefore convergent. ⇤

108
1
X nn
Example 118. Test the convergence of the series .
n=1
n!

Solution.
Since the term
nn Let an
vn-1
are positive, we have
nn
=

n!
(nare+ 1)n+1
n! 1im (n91)(n 1)" n! n

Im Ian 1Iunlutics am/nt)


+
+
an+1 n n! N(n + in
n
=

1)(n + 1)n n!
1 n!.n
=

· · n
-

= = +

an (n + 1)! nn (n + 1)n! n
-"m(1+ t)" =e1 (
=n + 1 1
)n = (1 + )n ! e as n ! 1
n n
Ratio Test, the series an is
divergent.
Since eBy> 1, the given series is divergent by the Ratio Test. ⇤
the

THE ROOT TEST

n
p P1
(i) If lim |an | = L < 1, then the series n=1 an is absolutely convergent (and therefore
n!1
convergent).
p p P
(ii) If lim n |an | = L > 1 or lim n |an | = 1, then the series 1n=1 an is divergent.
n!1 n!1

n
p
(iii) If lim |an | = 1, the Ratio Test is inconclusive.
n!1

pn
If lim |an | = 1, then part (iii) of the Root Test says that the test gives no information.
n!1 P
The series an could converge or diverge. (If L = 1 in the Ratio Test, don’t try the Root Test
because L will again be 1. And if L = 1 in the Root Test, don’t try the Ratio Test because it will
fail too.)

X1
2n + 3 n
Example 119. Test the convergence of the series ( ) .
n=1
3n + 2

Solution.

Let an (2n +3)n2n + 3n,1


=

an = ( )n
3n + 2

In"lans=am[Intl. GT
11m
p 2 + n3 a
n
|an | =
2n + 3 Im 2
= n+ 82 !3n < 1 n-a
-

= 1
3n + 2 3+ n 3 +2

Thus the given series converges by the Root Test. ⇤


So, by the Root Test, the series Ian is
convergent.

109
4.3.5 Summary of Tests

We have developed a variety of tests to determine convergence or divergence for an infinite series
of constants. Here is a summary of the tests we have considered.

1. The nth Term Test: Unless an ! 0, the series diverges.


P n
2. Geometric series: ar converges if |r| < 1.
P
3. p-series: 1/np converges if p > 1; otherwise it diverges.

4. Series with nonnegative terms: Try the Integral Test, Ratio Test, or Root Test. Try
comparing to a known series with the Comparison Test or the Limit Comparison Test.
P P
5. Series with some negative terms: Does |an | converge? If yes, so does an since
absolute convergence implies convergence.
P
6. Alternating series: an converges if the series satisfies the conditions of the Alternating
Series Test.

4.4 Power Series

Definition 24. A power series about x = 0 is a series of the form


1
X
cn xn = c0 + c1 x + c2 x2 + · · · + cn xn + . . .
n=0

A power series about x = a is a series of the form


1
X
cn (x a)n = c0 + c1 (x a) + c2 (x a)2 + · · · + cn (x a)n + . . .
n=0

in which the center a and the coefficients c0 , c1 , . . . , cn , . . . are constants.

1
X
Example 120. For what values of x is the series n!xn convergent?
n=0
"
Solution. Let an n!2
=
Unz, 1

We use the Ratio Test. If we let an denote the nth term of the series, then an = n!xn . If x 6= 0,
live anIam
we have
enticanI callin
lim |
an+1
| = lim |
(n + 1)!xn+1
a (n + 1) (n) 0 is
| = lim (n + 1)|x| = 1
=

n!1 an n!1 n!xn n!1


ByBy the Ratio Test, the series converges only when n 0 and diverges
=

the Ratio Test, the series diverges when x 6= 0. Thus the given series converges only when
x = 0. ⇤
for all of 0.

110
Given Zan
Summari
1)In an Im an to or
meto
an Δ the series [an divergent
by the Test for
Divergence
Iman 0
=

C) Zarr-(check an= arv-1) the Earn


-

convergent if Irk 1

divergent if Irk, 1

3) In(check an: i) the p-series ofp>/


convergent
divergent if p 1
-

If first 3 doesn't work.....

him an =
0
1-

10
(i) an30 Xnc, I (ii) an <O for (ii) an=1-1)"by or

5
some n

1 1)n 1bn
+

Ilant
converges? an = -

Integral test Does


·
·

* v ·

Direct Comparison ·

Absolutely convergent
·

Alternating series Test


Test
Ratio Test
av
=>
convergent
·Limit comparison &
(theorem 29) Root Test ·
Test
->
·Root Test

Ratio Test

MATH
P1 (x 3)n
Example 121. For what values of x does the series n=1 converge?
n
(n -
3)n
Solution. let an =

N
V-xxI

(x 3)n
him
Let I am
a n = inn each
. Then cul: ta(n-3).
) n
1a 3/n
=
-

111 1a 3)
= -

a (x 3)n+1 the n+1


n and hence convergent when
By the Ratio
| = test,
is
| absolutely| · convergentn |
series
an n+1 (x 3)
L:(x-31>1, and 1 (n-31> 1.
divergent when
= 1 |x 3| ! |x 3| as n ! 1
(n 3/4/(=) 1(n 3</(>
-
1 + n 24x4 - -

BySo,thethe
Ratioseries convergent
Test, the given serieswhen
is absolutely
2024convergent,
and and therefore convergent, when |x 3| <
1 and divergent when |x 3| > 1. Now
divergent when a 2 or n>4. When n 2,
=

3)20 (1".

(4
|x =
3| < 1 , 2<x<4
-

I b
Let
n
n1
=

sonim
the
b =
series converges
0
= when 2 <nt
P bat
and x <t 4= and
bn diverges when x < 2 or x > 4. If we put x = 4
=

in the
PSo, by series, it becomes 1/n, the harmonic series, which is divergent. If x = 2, the series is
the
Alternating series test, the series as
( 1) /n, which converges by the Alternating Series Test.convergent.
n
Thus the given power series converges
when
n=4,512-31-5
for 2  x < 4. ⇤ is a
pseries with
p =1,
so it is divergent.

So, the series


converges for C [n<4.

n 2n
#Example 122. For what values of x does the series P1 ( 1) x converge?
n=0 2n 2 2 (n!)
Solution.
( 1)n x2n 1 1)"
-
22n
Let an = . Then
22n (n!)2Let
an =
Vn<1
22(n!)"

I'm an in | tint
an+1
an
| aint
= |
22(n+1) [(ncarini
( 1)n+1 x2(n+1)
= | / 1..nonts.Iend
·
22n (n!)2
+ 1)!]2 ( 1)n x2n
((-1..n.i,- ) .mn
2n+2 2n
(n!)2
= re:
x vntR 2 =
-

2n+2
· 0
2 (n + 1)2 (n!)2 x2n
:By the Ratio Test, the series x2
for all at 1.
converges
= !0<1 for all x
4(n + 1)2

Thus, by the Ratio Test, the given series converges for all values of x. ⇤

P1
Theorem 30. For a given power series n=0 cn (x a)n , there are only three possibilities:

(i) The series converges only when x = a.

(ii) The series converges for all x.

(iii) There is a positive number R such that the series converges if |x a| < R and diverges if
|x a| > R.

111
The number R is called the radius of convergence of the power series. The interval of conver-
gence of a power series is the interval that consists of all values of x for which the series converges.
In case (i) the interval consists of just a single point a. In case (ii) the interval is ( 1, 1). In
case (iii) note that the inequality |x a| < R can be rewritten as a R < x < a + R. When x is
an endpoint of the interval, that is, x = a ± R, anything can happen – the series might converge
at one or both endpoints or it might diverge at both endpoints. Thus in case (iii) there are four
possibilities for the interval of convergence:

(a R, a + R) (a R, a + R] [a R, a + R) [a R, a + R]

We summarize here the radius and interval of convergence for each of the examples already con-
sidered.
In -al < r
>Roy,m<ap
a!p
(V1 < 1 => (x/<)

Series Radius of convergence Interval of convergence (x 0/s/ -

X1
Geometric Series 1x a|<k
xn
-

R=1 ( 1, 1)
a 0 x,
=
= 10/c I
n=0
1
X
Exp 120 n!xn R=0 {0}
n=0
1
X (x 3)n
Exp 121 R=1 [2, 4)
n=0
n
1
X ( 1)n x2n
R=1 ( 1, 1)
Exp 122 22n (n!)2
n=0

Example 123. Find the radius of convergence and interval of convergence of the series
X1
( 3)n xn
p
n=0
n+1

Solution. 1
-
3(r n
x

pan (n + Let =

V-n>, I
n n
Let an = ( 3) x / n + 1. Then 1

1.51
Im Iant1 Im /
a
=
(2 3)n+1 xn+1 enn +I 1:nmA31xII
p
=

·
p
n+1
= 3x
/n
r
+ 1 nIm
n= +
nmut 03(x)It=3(x)
an n+2 ( 3)n xn n+2
By the Ratio Test, the seriess is convergent if 31211 and divergent when 31x1>1
1 + (1/n)
31x1>/<=) ( 5.= 3 so, the radius |x| !of3|x|
<
as nR! 5.
convergence, 1 =

1 + (2/n)
5oina -2
(" Fl
Ei
his when n
1 1 1+
-
=

i
+

(n
=

n=
0
+
....
1
+
1
By the Ratio Test, the given series converges if 3|x| < 1 and diverges if 3|x| > 1. Thus it converges
if |x| > 13 . This means that the radius =-
divergent Iis isp Rseries
So the series is
if |x| < 13 and diverges = 13 . with p=
:.
of convergence a c |
1 1 1
We know the series converges in the interval ( 3 , 3 ). If x = 3 , the series becomes
(i) when n
13,
=

in
1
the E 1-3)(b)uEt
/n
1
=

n0
=
+ 1
= nb t

=
1 X ( 3) ( )
n 1 n X 1 1 1 1 1
by =
Mm0 0
=
3
pand but sn
= psn) bn
" p + p + p + p + ...
= vn. i

Sn+1 =

n=0
n+1 n=0
n+1 1 2 3 4
By the
Alternating Series Test, the series is
convergent.
: The interval of convergence is 1-13, s]
112

You might also like